diff --git a/optimierung/Uebungsblatt3.tex b/optimierung/Uebungsblatt3.tex new file mode 100644 index 0000000..7769b90 --- /dev/null +++ b/optimierung/Uebungsblatt3.tex @@ -0,0 +1,366 @@ +\documentclass[10pt,a4paper,oneside,ngerman,numbers=noenddot]{scrartcl} +\usepackage[T1]{fontenc} +\usepackage[utf8]{inputenc} +\usepackage[ngerman]{babel} +\usepackage{amsmath} +\usepackage{amsfonts} +\usepackage{amssymb} +\usepackage{paralist} +\usepackage{gauss} +\usepackage{pgfplots} +\usepackage[locale=DE,exponent-product=\cdot,detect-all]{siunitx} +\usepackage{tikz} +\usetikzlibrary{matrix,fadings,calc,positioning,decorations.pathreplacing,arrows,decorations.markings} +\usepackage{polynom} +\polyset{style=C, div=:,vars=x} +\pgfplotsset{compat=1.8} +\pagenumbering{arabic} +% ensures that paragraphs are separated by empty lines +\parskip 12pt plus 1pt minus 1pt +\parindent 0pt +% define how the sections are rendered +\def\thesection{\arabic{section})} +\def\thesubsection{\alph{subsection})} +\def\thesubsubsection{(\roman{subsubsection})} +% some matrix magic +\makeatletter +\renewcommand*\env@matrix[1][*\c@MaxMatrixCols c]{% + \hskip -\arraycolsep + \let\@ifnextchar\new@ifnextchar + \array{#1}} +\makeatother + +\begin{document} +\author{Jan Branitz (6326955), Jim Martens (6420323),\\ +Stephan Niendorf (6242417)} +\title{Hausaufgaben zum 4. November} +\maketitle +\section{} %1 + \subsection{} %a + \textbf{Aufgabe:} Lösen Sie das folgende LP-Problem mit dem Simplexverfahren: + \begin{alignat*}{3} + \text{maximiere}\; & x_{1} &+& 2x_{2} && \\ + \multicolumn{6}{l}{\text{unter den Nebenbedingungen}} && \\ + \;-& 4x_{1} &+& x_{2} &\leq & 1 \\ + \;-& x_{1} &+& x_{2} &\leq & 2 \\ + \;& \frac{1}{2}x_{1} &-& x_{2} &\leq & 1 \\ + \multicolumn{4}{r}{$x_{1}, x_{2}$} \,&\geq &\, 0 + \end{alignat*} + + \textbf{Lösung.} + + \underline{Starttableau}: + \begin{alignat*}{4} + x_{3} \,&=&\, 1 \,&+&\, 4x_{1} \,&-&\, x_{2} \\ + x_{4} \,&=&\, 2 \,&+&\, x_{1} \,&-&\, x_{2} \\ + x_{5} \,&=&\, 1 \,&-&\, \frac{1}{2}x_{1} \,&+&\, x_{2} \\ \cline{1 - 9} + z &=& && x_{1} \,&+&\, 2x_{2} + \end{alignat*} + + \underline{1. Iteration}: + + Eingangsvariable: $x_{2}$\\ + Ausgangsvariable: $x_{3}$ + + Es folgt + \begin{alignat*}{2} + x_{2} \,&=&&\, 1 + 4x_{1} - x_{3} \\ + x_{4} \,&=&&\, 2 + x_{1} - \left(1 + 4x_{1} - x_{3}\right) \\ + &=&&\, 2 + x_{1} - 1 - 4x_{1} + x_{3} \\ + &=&&\, 1 - 3x_{1} + x_{3} \\ + x_{5} \,&=&&\, 1 - \frac{1}{2}x_{1} + \left(1 + 4x_{1} - x_{3}\right) \\ + &=&&\, 1 - \frac{1}{2}x_{1} + 1 + 4x_{1} - x_{3} \\ + &=&&\, 2 + \frac{7}{2}x_{1} - x_{3} \\ + z \,&=&&\, x_{1} + 2\left(1 + 4x_{1} - x_{3}\right) \\ + &=&&\, x_{1} + 2 + 8x_{1} - 2x_{3} \\ + &=&&\, 2 + 9x_{1} - 2x_{3} + \end{alignat*} + + \underline{Ergebnis der 1. Iteration}: + \begin{alignat*}{4} + x_{2} \,&=&\, 1 \,&+&\, 4x_{1} \,&-&\, x_{3} \\ + x_{4} \,&=&\, 1 \,&-&\, 3x_{1} \,&+&\, x_{3} \\ + x_{5} \,&=&\, 2 \,&+&\, \frac{7}{2}x_{1} \,&-&\, x_{3} \\ \cline{1 - 9} + z &=& 2 \,&+&\, 9x_{1} &-& 2x_{3} + \end{alignat*} + + \underline{2. Iteration}: + + Eingangsvariable: $x_{1}$ \\ + Ausgangsvariable: $x_{4}$ + + Es folgt + \begin{alignat*}{2} + 3x_{1} &=&& 1 + x_{3} - x_{4} \\ + x_{1} &=&& \frac{1}{3} + \frac{1}{3}x_{3} - \frac{1}{3}x_{4} \\ + x_{2} &=&& 1 + 4\left(\frac{1}{3} + \frac{1}{3}x_{3} - \frac{1}{3}x_{4}\right) - x_{3} \\ + &=&& 1 + \frac{4}{3} + \frac{4}{3}x_{3} - \frac{4}{3}x_{4} \\ + &=&& \frac{7}{3} + \frac{4}{3}x_{3} - \frac{4}{3}x_{4} \\ + x_{5} &=&& 3 + \frac{7}{2}\left(\frac{1}{3} + \frac{1}{3}x_{3} - \frac{1}{3}x_{4}\right) - x_{3} \\ + &=&& 3 + \frac{7}{6} + \frac{7}{6}x_{3} - \frac{7}{6}x_{4} - x_{3} \\ + &=&& \frac{13}{6} + \frac{1}{6}x_{3} - \frac{7}{6}x_{4} \\ + z &=&& 2 + 9\left(\frac{1}{3} + \frac{1}{3}x_{3} - \frac{1}{3}x_{4}\right) - 2x_{3} \\ + &=&& 2 + 3 + 3x_{3} - 3x_{4} - 2x_{3} \\ + &=&& 5 + x_{3} - 3x_{4} + \end{alignat*} + + \underline{Ergebnis der 2. Iteration}: + \begin{alignat*}{4} + x_{1} \,&=&\, \frac{1}{3} \,&+&\, \frac{1}{3}x_{3} \,&-&\, \frac{1}{3}x_{4} \\ + x_{2} \,&=&\, \frac{7}{3} \,&+&\, \frac{4}{3}x_{3} \,&-&\, \frac{4}{3}x_{4} \\ + x_{5} \,&=&\, \frac{13}{6} \,&+&\, \frac{1}{6}x_{3} \,&-&\, \frac{7}{6}x_{4} \\ \cline{1 - 7} + z &=& 5 \,&+&\, x_{3} \,&-&\, 3x_{4} + \end{alignat*} + + \underline{3. Iteration}: + + Eingangsvariable: $x_{3}$ \\ + Ausgangsvariable: keine vorhanden + + Es gibt keine optimale Lösung, da es keine Basisvariable gibt, die $x_{3}$ beschränkt. Damit ist dieses Problem unbeschränkt. + + \underline{Startlösung ("`zulässige Basislösung am Anfang"')}: + \[ + x_{1} = 0, x_{2} = 0, x_{3} = 1, x_{4} = 2, x_{5} = 1 \text{ mit } z = 0 + \] + \underline{Zulässige Basislösung nach der 1. Iteration}: + \[ + x_{1} = 0, x_{2} = 1, x_{3} = 0, x_{4} = 1, x_{5} = 2 \text{ mit } z = 2 + \] + \underline{Zulässige Basislösung nach der 2. Iteration}: + \[ + x_{1} = \frac{1}{3}, x_{2} = \frac{7}{3}, x_{3} = 0, x_{4} = 0, x_{5} = \frac{13}{6} \text{ mit } z = 5 + \] + + Ermittlung einer Halbgeraden des $\mathbb{R}^{2}$: + + \begin{alignat*}{2} + x_{3} &=& t \\ + x_{4} &=& 0 \\ + x_{1} &=& \frac{1}{3} + \frac{1}{3}t \\ + x_{2} &=& \frac{7}{3} + \frac{4}{3}t \\ + x_{5} &=& \frac{13}{6} + \frac{1}{6}t \\ + z &=& 5 + t \\ + t &\geq & 0 + \end{alignat*} + + Daraus ergibt sich in Parameterform: + \begin{alignat*}{2} + (x_{1}, x_{2}) &=& \left(\frac{1}{3} + \frac{1}{3}t, \frac{7}{3} + \frac{4}{3}t\right) \\ + &=& \left(\frac{1}{3}, \frac{7}{3}\right) + t\left(\frac{1}{3}, \frac{4}{3}\right) + \end{alignat*} + \begin{alignat*}{2} + (x_{3}, x_{2}) &=& \left(t, \frac{7}{3} + \frac{4}{3}t\right) \\ + &=& \left(0, \frac{7}{3}\right) + t\left(1, \frac{4}{3}\right) + \end{alignat*} + + Da in diesem Fall $x_{1}$ eine Basisvariable ist und somit nicht gleich $t$ ist, stellt $t$ eine beliebige positive Konstante dar. Daher verändert sich auch die Gerade je nach Wahl von $t$. Deswegen ist es nicht möglich genau eine Halbgerade zu finden, auf der die Zielfunktion beliebig große Werte annimmt. + \subsection{} %b + Durch Umstellen der Nebenbedingungen des Problems aus a nach $x_{2}$ ergibt sich: + \begin{alignat*}{3} + x_{2} &\leq & 4x_{1} &+& 1 \\ + x_{2} &\leq & x_{1} &+& 2 \\ + x_{2} &\geq & \frac{1}{2}x_{1} &-& 1 + \end{alignat*} + Daraus lässt sich die Fläche aller gültigen Werte zeichnen. + + \begin{tikzpicture}[>=stealth] + \begin{axis}[ + ymin=0,ymax=7, + x=1cm, + y=1cm, + axis x line=middle, + axis y line=middle, + axis line style=->, + xlabel={$x_{1}$}, + ylabel={$x_{2}$}, + xmin=0,xmax=7 + ] + + \addplot[no marks, black, -] expression[domain=0:6,samples=100]{4*x + 1} node[pos=0.65,anchor=north]{}; + \addplot[no marks, black, -] expression[domain=0:6,samples=100]{1*x + 2} node[pos=0.65,anchor=north]{}; + \addplot[no marks, black, -] expression[domain=0:6,samples=100]{0.5*x - 1} node[pos=0.65,anchor=north]{}; + \addplot[no marks, black, -] expression[domain=0:6,samples=100]{1.333333333333333*x + 2.33333333333333333} node[pos=0.65,anchor=north]{}; + %\node at (axis cs: 2.5,4.5) {(2.25,3.75)}; + %\node at (axis cs: 6,2) {z}; + %\draw[>=stealth] (axis cs:1,0) -- (axis cs:1,-6) node [pos=0.65,anchor=north]{}; + \end{axis} + \end{tikzpicture}\\ +\section{} %2 + \subsection{} %a + \textbf{Aufgabe:} Lösen Sie das folgende LP-Hilfsproblem mit dem Simplexverfahren: + \begin{alignat*}{5} + \text{maximiere}\; -& x_{0} && && && && \\ + \multicolumn{10}{l}{\text{unter den Nebenbedingungen}} && \\ + \; & &-& x_{1} &-& x_{2} &-& x_{0} &\leq & -4 \\ + \; &&& x_{1} &+& 2x_{2} &-& x_{0} &\leq & 2 \\ + \; &&-&x_{1} &+& x_{2} &-& x_{0} &\leq & 1 \\ + \multicolumn{8}{r}{$x_{0}, x_{1}, x_{2}$} \,&\geq &\, 0 + \end{alignat*} + + \textbf{Lösung.} + + \underline{Starttableau}: + \begin{alignat*}{5} + x_{3} \,&=&\, -4 \,&-&\, x_{1} \,&+&\, x_{2} \,&+&\, x_{0} \\ + x_{4} \,&=&\, 2 \,&-&\, x_{1} \,&-&\, 2x_{2} \,&+&\, x_{0} \\ + x_{5} \,&=&\, 1 \,&+&\, x_{1} \,&-&\, x_{2} \,&+&\, x_{0} \\ \cline{1 - 9} + w &=& && && \,&-&\, x_{0} + \end{alignat*} + + Umwandeln in ein zulässiges Tableau: + + Eingangsvariable: $x_{0}$\\ + Ausgangsvariable: $x_{3}$ + + Es folgt + \begin{alignat*}{2} + -x_{0} \,&=&&\, -4 - x_{1} + x_{2} - x_{3} \\ + x_{0} \,&=&&\, 4 + x_{1} - x_{2} + x_{3} \\ + x_{4} \,&=&&\, 2 - x_{1} - 2x_{2} + \left(4 + x_{1} - x_{2} + x_{3}\right) \\ + &=&&\, 2 - x_{1} - 2x_{2} + 4 + x_{1} - x_{2} + x_{3} \\ + &=&&\, 6 - 3x_{2} + x_{3} \\ + x_{5} \,&=&&\, 1 + x_{1} - x_{2} + \left(4 + x_{1} - x_{2} + x_{3}\right) \\ + &=&&\, 1 + x_{1} - x_{2} + 4 + x_{1} - x_{2} + x_{3} \\ + &=&&\, 5 + 2x_{1} - 2x_{2} + x_{3} \\ + w \,&=&&\, -\left(4 + x_{1} - x_{2} + x_{3}\right) \\ + &=&&\, -4 - x_{1} + x_{2} - x_{3} \\ + \end{alignat*} + + \underline{Ergebnis des Umwandelns}: + \begin{alignat*}{5} + x_{0} \,&=&\, 4 \,&+&\, x_{1} \,&-&\, x_{2} \,&+&\, x_{3} \\ + x_{4} \,&=&\, 6 \,&& &-&\, 3x_{2} \,&+&\, x_{3} \\ + x_{5} \,&=&\, 5 \,&+&\, 2x_{1} \,&-&\, 2x_{2} \,&+&\, x_{3} \\ \cline{1 - 9} + w &=& -2 \,&-&\, x_{1} \,&+&\, x_{2} \,&-&\, x_{3} + \end{alignat*} + + \underline{1. Iteration}: + + Eingangsvariable: $x_{2}$ \\ + Ausgangsvariable: $x_{4}$ + + Es folgt + \begin{alignat*}{2} + 3x_{2} &=&& 6 + x_{3} - x_{4} \\ + x_{2} &=&& 2 + \frac{1}{3}x_{3} - \frac{1}{3}x_{4} \\ + x_{0} &=&& 4 + x_{1} - \left(2 + \frac{1}{3}x_{3} - \frac{1}{3}x_{4}\right) + x_{3} \\ + &=&& 4 + x_{1} - 2 - \frac{1}{3}x_{3} + \frac{1}{3}x_{4} + x_{3}\\ + &=&& 2 + x_{1} + \frac{2}{3}x_{3} + \frac{1}{3}x_{4} \\ + x_{5} &=&& 5 + 2x_{1} - 2\left(2 + \frac{1}{3}x_{3} - \frac{1}{3}x_{4}\right) + x_{3} \\ + &=&& 5 + 2x_{1} - 4 - \frac{2}{3}x_{3} + \frac{2}{3}x_{4} + x_{3} \\ + &=&& 1 + 2x_{1} + \frac{1}{3}x_{3} + \frac{2}{3}x_{4} \\ + w &=&& -2 - x_{1} + \left(2 + \frac{1}{3}x_{3} - \frac{1}{3}x_{4}\right) - x_{3} \\ + &=&& -2 - x_{1} + 2 + \frac{1}{3}x_{3} - \frac{1}{3}x_{4} + x_{3} \\ + &=&& 0 - x_{1} + \frac{4}{3}x_{3} - \frac{1}{3}x_{4} + \end{alignat*} + + \underline{Ergebnis der 1. Iteration}: + \begin{alignat*}{5} + x_{2} \,&=&\, 2 \,&& &+&\, \frac{1}{3}x_{3} \,&-&\, \frac{1}{3}x_{4} \\ + x_{0} \,&=&\, 2 \,&+&\, x_{1} \,&+&\, \frac{2}{3}x_{3} \,&+&\, \frac{1}{3}x_{4} \\ + x_{5} \,&=&\, 1 \,&+&\, 2x_{1} \,&+&\, \frac{1}{3}x_{3} \,&+&\, \frac{2}{3}x_{4} \\ \cline{1 - 9} + w &=& 0 \,&-&\, x_{1} \,&+&\, \frac{4}{3}x_{3} \,&-&\, \frac{1}{3}x_{4} + \end{alignat*} + + Da das Hilfsproblem keine optimale Lösung besitzt, besitzt das ursprüngliche Problem keine zulässige Lösung und ist damit unlösbar. + + \subsection{} %b + \textbf{Aufgabe:} Lösen Sie das folgende LP-Hilfsproblem mit dem Simplexverfahren: + \begin{alignat*}{5} + \text{maximiere}\; -& x_{0} && && && && \\ + \multicolumn{10}{l}{\text{unter den Nebenbedingungen}} && \\ + \; & && x_{1} &-& x_{2} &-& x_{0} &\leq & 8 \\ + \; & &-& x_{1} &-& x_{2} &-& x_{0} &\leq & -3 \\ + \; & &-& x_{1} &+& 4x_{2} &-& x_{0} &\leq & 2 \\ + \multicolumn{8}{r}{$x_{0}, x_{1}, x_{2}$} \,&\geq &\, 0 + \end{alignat*} + + \textbf{Lösung.} + + \underline{Starttableau}: + \begin{alignat*}{5} + x_{3} \,&=&\, 9 \,&-&\, x_{1} \,&+&\, x_{2} \,&+&\, x_{0} \\ + x_{4} \,&=&\, -3 \,&+&\, x_{1} \,&+&\, x_{2} \,&+&\, x_{0} \\ + x_{5} \,&=&\, 2 \,&+&\, x_{1} \,&-&\, 4x_{2} \,&+&\, x_{0} \\ \cline{1 - 9} + w &=& && && \,&-&\, x_{0} + \end{alignat*} + + Umwandeln in ein zulässiges Tableau: + + Eingangsvariable: $x_{0}$\\ + Ausgangsvariable: $x_{4}$ + + Es folgt + \begin{alignat*}{2} + -x_{0} \,&=&&\, -3 + x_{1} + x_{2} - x_{4} \\ + x_{0} \,&=&&\, 3 - x_{1} - x_{2} + x_{4} \\ + x_{3} \,&=&&\, 9 - x_{1} + x_{2} + \left(3 - x_{1} - x_{2} + x_{4}\right) \\ + &=&&\, 9 - x_{1} + x_{2} + 3 - x_{1} - x_{2} + x_{4} \\ + &=&&\, 12 - 2x_{1} + x_{4} \\ + x_{5} \,&=&&\, 2 + x_{1} - 4x_{2} + \left(3 - x_{1} - x_{2} + x_{4}\right) \\ + &=&&\, 2 + x_{1} - 4x_{2} + 3 - x_{1} - x_{2} + x_{4} \\ + &=&&\, 5 - 5x_{2} + x_{4} \\ + w \,&=&&\, -\left(3 - x_{1} - x_{2} + x_{4}\right) \\ + &=&&\, -3 + x_{1} + x_{2} - x_{4} \\ + \end{alignat*} + + \underline{Ergebnis des Umwandelns}: + \begin{alignat*}{5} + x_{0} \,&=&\, 3 \,&-&\, x_{1} \,&-&\, x_{2} \,&+&\, x_{4} \\ + x_{3} \,&=&\, 12 \,&-&\, 2x_{1} \,&& &+&\, x_{4} \\ + x_{5} \,&=&\, 5 \,&& &-&\, 5x_{2} \,&+&\, x_{4} \\ \cline{1 - 9} + w &=& -3 \,&+&\, x_{1} \,&+&\, x_{2} \,&-&\, x_{4} + \end{alignat*} + + \underline{1. Iteration}: + + Eingangsvariable: $x_{1}$ \\ + Ausgangsvariable: $x_{0}$ + + Es folgt + \begin{alignat*}{2} + x_{1} &=&& 3 - x_{2} + x_{4} - x_{0} \\ + x_{3} &=&& 12 - 2\left(3 - x_{2} + x_{4} - x_{0}\right) + x_{4} \\ + &=&& 12 - 6 + 2x_{2} - 2x_{4} + 2x_{0} + x_{4} \\ + &=&& 6 + 2x_{2} - x_{4} + 2x_{0} \\ + x_{5} &=&& 5 - 5x_{2} + x_{4} \\ + w &=&& -3 + \left(3 - x_{2} + x_{4} - x_{0}\right) + x_{2} - x_{4} \\ + &=&& -3 + 3 - x_{2} + x_{4} - x_{0} + x_{2} - x_{4} \\ + &=&& - x_{0} + \end{alignat*} + + \underline{Ergebnis der 1. Iteration}: + \begin{alignat*}{5} + x_{1} \,&=&\, 3 \,&-&\, x_{2} \,&+&\, x_{4} \,&-&\, x_{0} \\ + x_{3} \,&=&\, 6 \,&+&\, 2x_{2} \,&-&\, x_{4} \,&+&\, 2x_{0} \\ + x_{5} \,&=&\, 5 \,&-&\, 5x_{2} \,&-&\, x_{4} \,&& \\ \cline{1 - 9} + w &=& && && &-& x_{0} + \end{alignat*} + + Das Tableau ist optimal. Als optimale Lösung des Hilfsproblem erhält man: + \[ + x_{0} =0, x_{1} = 3, x_{2} = 0 + \] + + Als zulässige Lösung für das ursprüngliche Problem ergibt sich: + \[ + x_{1} = 3, x_{2} = 0 + \] + + Die ursprüngliche Zielfunktion lautet $z = x_{1} + 3x_{2}$. Setzt man für $x_{1}$ die rechte Seite der Gleichung im obigen Tableau ein, erhält man: + + \[ + z = 3 - x_{2} + x_{4} + 3x_{2} = 3 + 2x_{2} + x_{4} + \] + + Daraus ergibt sich dieses Starttableau: + + \begin{alignat*}{4} + x_{1} \,&=&\, 3 \,&-&\, x_{2} \,&+&\, x_{4} \\ + x_{3} \,&=&\, 6 \,&+&\, 2x_{2} \,&-&\, x_{4} \\ + x_{5} \,&=&\, 5 \,&-&\, 5x_{2} \,&-&\, x_{4} \\ \cline{1 - 7} + z \,&=&\, 3 \,&+&\, 2x_{2} \,&+&\, x_{4} + \end{alignat*} + +\end{document}